« first day (29 days earlier)      last day (545 days later) » 
00:00 - 16:0016:00 - 00:00

12:02 AM
0
Q: Let $A \subseteq \mathbb R^n$ and $|A| = n$ for some $n \in \mathbb N.$ Prove that the convex hull of $A$ is the smallest convex set containing $A$.

VladimirI am aware that there are similar threads to this, but since I am very new to this sort of math, I couldn't make sense of them too much. Here's a proof(?): Suppose $B$ is a convex set and $C$ is a convex hull of $A$ such that $A \subseteq B \subseteq C \subseteq \mathbb R^n.$ If $A = \{v_1, v_...

0
Q: real analysis problem continuous function

user294794Consider a function f(x) ; R → R such that limx→∞= +∞ and limx→-∞= -∞. Show that the range of this function is R.

Title contains problem. Short question. real analysis problem continuous function
0
Q: Moment generating funtion of gamma random variable

melissahow would I find the mgf of a gamma rv with parameter k > 0 and λ > 0 ?

0
Q: Proving function of random variable is a random variable?

majmunFor information on the question, see This question: I wanted to add a more measure-theoretic answer (I still do, but I can just link this question there if this is answered well). However, when proving the question I confused myself, so I want someone to tell me if this is correct, or how to bet...

0
Q: Continuous cumulative distribution function

JohnHow can you show a function is a cumulative distribution function and then how can you show this cdf describes a continuous random variable?

 
12:21 AM
0
Q: Please check my proof that a collection of connected subsets is connected

ClaycrusherThe problem reads: Let $\{ E_j : j\in J \}$ be a collection of connected subsets of a metric space $X$ such that $E_i \cap E_j \neq \emptyset$ for every $i,j\in J$. Prove that $\cup _{j\in J} E_j$ is connected. Before marking this a duplicate, please consider that I am aware that there are...

 
0
Q: Etiquette about posting a large number of questions about preliminary exams.

KrampusI am Currently studying for my preliminary exams (Advanced Calculus and Linear Algebra), and I can solve about half of the previous years exam questions. However, I do not have anyone to vet my proofs at the moment. My question is it appropriate to ask a large number of questions on here?

 
0
Q: Prove: A set has a partial-order relation R on it. P is a subset of this set. Prove that the restriction of R to P is itself a partial-order relation.

smitty_werbermanjensenAssume that this relation, T, resulting from the restriction of R on P is defined as such: T: (x,y) x ∈ P, Y ∈ P {(x, y) ∈ R}. It seems to be obvious that if R works on A, and B contains some subset of A, then T (R on B) is also a partial-order relation; that, or B is the non-orderable units of...

0
Q: Problem 12.22 on Analytic Semigroups In Renardy's An Introduction to Partial Differential Equations

User112358I am trying to answer question 12.22 In this book: http://uxmym1.iimas.unam.mx/ramon/docs/RenRog.pdf Here is the question: Verify that for $n\in\mathbb{N},$ we have $(-A)^{-n}=(-A^{-1})^n,$ where $(-A)^{-n}$ is defined by: $$ (-A)^{-\alpha}=-\frac{1}{2\pi i} \int_{\Gamma} \lambda^{-\alpha}...

 
0
Q: Should this question be protected?

LebI saw that this question was protected and the accepted answer referred to another older post which wasn't. At a minimum, shouldn't the older one be protected as well? Also, shouldn't the newer one be possibly marked as a duplicate? While they differ at a first glance, they are really talking ...

 
0
Q: Integral $\int_0^\infty\operatorname{arccot}(x)\,\operatorname{arccot}(2x)\,\operatorname{arccot}(5x)\,dx$

Piotr ShatalinI have to evaluate this definite integral: $$Z=\int_0^\infty\operatorname{arccot}(x)\,\operatorname{arccot}(2x)\,\operatorname{arccot}(5x)\,dx$$ My CAS was only able to find its approximate numeric value: $$Z\approx0.796300956669079523165601562454031588576893734085453548868394...$$ Is there an ap...

0
Q: Let X_1,...,X_n be a random sample from the Pareto(γ) distribution. Find the UMVUE for γ and the UMVUE for 1/γ

jmw_codeI need to find the UMVUE for γ and for 1/γ but am lost on how. I'm having trouble finding a complete and sufficient statistic so I can't begin to find the UMVUE.

0
Q: Order statistics excercise

FulaphexI have trouble solving this task: Let $Y_1 < Y_2 < Y_3$ denote the order statistics of a random sample of size 3 from a distribution with pdf $f(x) = 1$, $0 < x < 1$, zero elsewhere. Let $$Z = \frac{(Y_1 + Y_3)}{2}$$ be the midrange of the sample. Find the pdf of $Z$. So far I have got this: ...

0
Q: Related Rates - Ships

Casey FosterShip A is currently 85 km south of ship B. Ship A travels north at 30 km/h and ship B travels east at 20 km/h. How fast is the distance between them changing in 1.5 hours? I have established the givens but I'm not completely sure how to proceed with this related rates problem. Do I use Pythagora...

 
12:51 AM
0
Q: How to determine change of basis matrix?

Glenn2329How do you find the change of basis matrix from a Basis to a standard basis for a 2x2 matrix A?

 
0
Q: Why didn'd i get Necromancer badge?

Георгий ЛанецTrying to get Necromancer badge i answered this question. Why i have not badge? This question is asked in 2009 year. Maybe Answer means answer marked as answer?

 
0
Q: Edelstein Fixed Point Theorem

MariaULet $(M,d)$ be a metric space, $M$ compact. If $f:M->M$ is contractive [ ie $d(f(x), f(y)) < d(x,y) , \forall x,y \in M$] then $\exists x_0 \in M $ unique s.t $f(x_0)=x_0$ . Sugestion: $g: M -> R$ with $g(x)=d(x, f(x)) $ is continuous relative to $d$. I was able to prove the continuity of g, b...

0
Q: Prove that the tangent at $P$ intersects $C$ twice at $P$ and once at $4P$; the tangent at $5P$ intersects $C$ twice at $5P$ and once at $2P$.

Johanne Marie ChiuThis is a problem from Conics and Cubics by Bix. Please help me answer this one. Let $C$ be a nonsingular, irreducible cubic with a flex $O$. Add points (commutative) of $C$ with respect to $O$ as base point. Let $P$ be a point on $C$ of order 6. Prove that the tangent at $P$ intersects $C$ twic...

0
Q: Prooof by involving distint prime numbers

RexProve 3+ 5(sqrt 2) is irrational. I have some ideas about this proof , but I am not quite finished. I undertand being irrational mean the number would not be in the form of p/q. I have proved root 2 is irrational before, but am a bit confused with this one, any ideas. Thank You in advance.

Tag (proof-writing) should not be the only tag a question has. Please add a tag for a subject area to which the question belongs. (autocomment)Normal Human 21 secs ago
0
Q: How do I do a finite expansion of \inte^x^2?

user294826I am looking for a way to make a finite expression of the integral of e to the x-squared. How can I go about this? \int e^x^2\

0
Q: Pumping lemma for context free. How do I define the string 'w' and define cases?

Samserso DeskansonI am new to the pumping lemma for context free grammars. I have read books and researched online about the pumping lemma, however I am finding it difficult to understand the actual concept and how to determine all the case studies. A language that I want to use the pumping lemma for is R = {a^2j...

Welcome to Math.SE, Samserso Deskanson. This site uses MathJax formatting of formulas. More tips here. (autocomment)Normal Human 21 secs ago
0
Q: Normalisation of a range of numbers to another range

JuwairiaI want to normalise a set of range of values having 0 Min and a Max that is known but can vary; say 22000 and would like to normalise these values from 0 to 300 and also from 0 to 20. I found a formula here on stack Exchange that is a general formula. The general one-line formula to linearly res...

Welcome to Math.SE, Juwairia. This site uses MathJax formatting of formulas. More tips here. (from a bot)Normal Human 21 secs ago
0
Q: Finding a general formula for a regular expression

KadinI need to find the general formula for the regular expression (S+T)n where S and T are arbitrary regular expressions over a one-letter alphabet and n is an arbitrary natural The general formula for (X+Y)n where X and Y are arbitrary numbers is easy in that it just follows the formula xn + C(n, ...

Welcome to Math.SE, Kadin. This site uses MathJax formatting of formulas. More tips here. (from a bot)Normal Human 21 secs ago
0
Q: Euler differential equations

MikeI have following three equations $$ u'' - 2u = -2v$$ $$ u(0)=0 $$ $$ u'(1)=0 $$ and from these 3 equations I am trying to find u(v). It looks to me "Cauchy-Euler Differential Equations - Nonhomogeneous case" but I am not sure about that because it is not an exactly Cauchy form. Could you help m...

 
1:34 AM
0
Q: Before and after training running times

AstagThe mid-distance running coach, Zdravko Popovich, for the Olympic team of an eastern European country claims that his six-month training program significantly reduces the average time to complete a 1500-meter run. Four mid-distance runners were randomly selected before they were trained with coach...

This site uses MathJax formatting of formulas. More tips here. (autocomment)Normal Human 20 secs ago
0
Q: Setting up Iterated Integral to polar form

marg_ocruzSetup the integral $$\int\int_D f(x,y) dA$$ as an iterated integral in polar coordinates where $$D=\left\{ (x,y)\vert |x|\leq y \leq 3, -3\leq x \leq 3\right\}$$. So I got the integral $$\int_{\frac{-\pi}{4}}^{\frac{\pi}{4}}\int_{|-3\sec\theta|}^{3\sqrt{2}} f(r\cos\theta, r\sin\theta) rdrd\theta...

0
Q: Definition of Work Done

JoeI am trying to make sense of the meaning of the definition of work. The original definition of work was also known as "the weight lifted through a height." I was hoping that our mathematical definition of work would correspond to this notion in a well-defined way but it seems like it doesn't. Co...

0
Q: Is this result for entire functions with no zeroes ... more basic than the Hadamard canonical product representation?

Lebron JamesI just worked on a problem and was able to solve it for the most part and also pretty easily, using Hadamard's product representation. But I wonder whether the solution that I compared my work to doesn't actually use the Hadamard factorization. The question asks to determine the class of all fu...

0
Q: What's the most complicated actually used equation you've seen?

mavaviljWhat's the most complicated actually used (i.e. it has some application and is not "purposely designed to be complicated") equation you've seen in mathematics?

Tag (soft-question) should not be the only tag a question has. Please add a tag for a subject area to which the question belongs. (from a bot)Normal Human 21 secs ago
0
Q: Nowhere dense subset of L1

Yuanchu DangWhy is $B_n = \{f \in L_1 : \int |f|^2 < n \}$ a nowhere dense subset of $L_1$? Please provide a proof without assuming that $L_2$ is a proper subset of $L_1$.

Short title. Title ends with a digit. Short question. Question contains please. Nowhere dense subset of L1
0
Q: How to prove Xn converges as n--> infinity

Spencer IrelandLet $x_n=(\sum_{k=1}^{n} \frac{1}{k})-logn$. Prove that $x_n$ converges as n-->$\infty$.

 
1:53 AM
Hello Normal Human
 
0
Q: Find all values of n

user19405892Let $n$ be an integer. Find all values of $n$ for which the Diophantine equation $n=a^2-b^2$ has a solution for integers $a$ and $b$. For those values of $n$ found in the previous part find all solutions of $n=a^2-b^2$ for integers $a$ and $b$. The first part is pretty easy. It is just if $n$ ...

Short title. Find all values of n
0
Q: a sum of random variables

john alexlet be $Z$ random variable, $Y_i$ iid random variable, $E(Y_i)=\mu$ , $var(Y_i)=\sigma^2$; $Z$ and $Y_i$ are independent. show that $E(X^2|Z)=\mu^2Z^2+\sigma^2Z$. I compute $E(X^2|Z=n)=nE(Y^2)+n\mu^2$ and i remplace the quantity $E(Y_i^2)$ by $var(Y)+(E(Y))^2=\sigma^2+\mu^2$, but i dont get th...

 
2:10 AM
0
Q: Efficient algorithm for optimization problem

user98235I had an interesting interview problem today. Let's assume that we have n boxes, containing many numbers. For instance, let's say n=4, and four boxes contain the following numbers: first box - (3, 2, 5) sum(first box) = 10 second box - (1, 7, 4, 8) sum(second box) = 20 t...

0
Q: Sigma notation calculus, how to answer without calculator

Valentina VacaHow do I answer this without using a calculator? 20 Σ x=1 (8j^3)

0
Q: How many ways are there to form a soccer team of 11 players from 12 boys and 14 girls if ...

Jimmy Tan1)the team has six boys and five girls? 2)there are at least one boy and at least one girl on the team?

0
Q: Discrete Math: Selecting/sorting

toad9In how many ways, can one select n letters from an unlimited supply of A’s, B’s and C’s so that there is an even number of A’s?

Short title. Short question. Discrete Math: Selecting/sorting
 
2:32 AM
0
Q: Show that C^2 ([0,1]) is not complete with respect to the C^1 norm

Spencer IrelandI want to show that $C^2$([0,1]) is not complete with respect to the $C^1$ norm. Recall that $||f||_{C^1}$ = $||f||_\infty$+$||f'||_\infty$.

Short question. [Show that C^2 ([0,1]) is not complete with respect to the C^1 norm](math.stackexchange.com/q/1554148)
0
Q: Volume of solid $W$ delimited by $z=x^2+3y^2$ and $z=8-x^2-y^2$

João PedroI need to solve this with triple integrals, however I'm having a lot of problems. Anyone knows how to solve?

0
Q: Probability of alarm being valid (bayes theorem)

mufcI have the following problem: your box is achieving a false positive rate of 0:01 and a false negative rate of 0:001. What fraction of the alarms that your box generates are valid alarms? I am trying to figure this out, but I am having trouble understanding how to set this problem up. My initi...

Consider adding a tag for a broader subject area to which the question belongs. Some of these tags might fit. (autocomment)Normal Human 20 secs ago
0
Q: Why I do not have Linear Algebra roots in my heart?

richieqianleI am not sure if this is the proper place to ask such a question. If not, I will move it or delete it. I have learned Linear Algebra several times. Every time I learn it, I always feel that: wow, it is beautiful. However, whenever I need to solve a problem related with it several months later, l...

Please don't use (self-learning) tag just because you were self-studying. This tag is only for questions about the process of self-studying. (from a bot)Normal Human 20 secs ago
0
Q: Question on Holder continuity with exponent 1/2 and its application

NinaninaI'm having a bit of trouble with the following problem: Suppose $f$ is square integrable on \mathbb{R}. Define $$F(x) = \int_0^x f(t) dt$$ Show that $F$ is Holder continuous with exponent 1/2. I tried to prove it directly by starting with plugging in $F$ into $|F(x) - F(y)|$ but got nowhere...

0
Q: I need to create a formula to help justify how I use my Budget at work

GhostOutlawSo I have a unique situation at work I'd like to create a formula or set of formulas for. I think it's an optimization and a carry capacity but I want to create the formula so I can justify to my boss why I did what I did or why I screwed up. So every month my company gets paid. We get paid ou...

0
Q: Evaluating a double integral with tricky limits.

HMPARTICLE$$\int^{2}_{0}\int^{0}_{-\sqrt{1-(y-1)^2}}xy^2dxdy = -0.8$$ I have evaluate this by changing to polar coordinates. I have sketched it and decided that $$\int^{0}_{-1}\int^{3\pi/2}_{\pi/2}r^4 \cos\theta \sin^2\theta d\theta dr = -2/15$$ but as you can see my answer differs from what is needed....

0
Q: killing vector field

VincentWhat is the definition of killing field?. The one my professor told me is : a smooth vector field V on M is called a Killing vector field for g if the flow of V acts by isometries of g. So what does it mean by the flow of V acts by isometries? I suspect this definition is equivalent to saying ...

Short title. killing vector field
 
2:59 AM
0
Q: Are "Verify if I'm correct" questions really on-topic?

Aloizio MacedoThere is a considerable amount of questions of the type Can you see if my proof/solution is correct? My question is simple: are they on-topic? My point is two-fold: I think they fail usefulness (and even create confusion) and may not be on-topic. With regards to the first aspect, sometime...

 
0
Q: distance from circumcentre to orthocentre

Sarnavo SarkarLet there be a triangle with angle $A,B,C$ Prove that the distance from circumcentre and orthocentre is $whole square root of (1-8cosAcosBcosC)

0
Q: Solving a differential equation by separating variables

Saagar GodithiHow can one solve the following differential equation by the technique of separation of variables? $$\frac{1}{x^2}\frac{dy}{dx}=y^5 \text{, when, }y(0)=-1$$

0
Q: Determining if a function is continuous at 0.

Matt B. How do I determine whether this function is continuous at x = 0?

 
3:24 AM
0
Q: Does there exist an online solution to the problem of a chain slipping off the edge of a table?

Mike JonesThe only reference I can find is in the preview of a textbook, namely, p.75 of Differential Equations with Boundary-Value Problems.

Tag (reference-request) should not be the only tag a question has. Please add a tag for a subject area to which the question belongs. (autocomment)Normal Human 21 secs ago
0
Q: A simple complex rational function that leads to a difficult question.

NilanLet $$f(z)=\dfrac{z-a}{z-b},\,\,\,\,\,\,z\not=b$$ be a complex valued rational function. How can I show that, if $|a|,|b|\lt1,$ then there is a complex number $z_0$ satisfying $|z_0|=1$ and $f(z_0)\in\mathbb{R}$ ? I have tried in many ways, but on success. Basically I tried to show that there...

 
3:38 AM
0
Q: Calculus (Limits) Question

Kushashwa Ravi ShrimaliFollowing is the question I've been trying to work on but can't get enough of it: $$\lim_{n\rightarrow \infty} \sin\left(\cfrac{n}{n^2+1^2}\right) + \sin\left(\cfrac{n}{n^2+2^2}\right) + \cdots + \sin\left({\cfrac{n}{n^2+n^2}}\right) $$ I'm required to find the value of the above limit. Al...

Words such as question are uninformative in titles. Please edit the title so that it better describes the specifics of your question. Do not hesitate to make it longer or include a formula if needed. More tips here. (from a bot)Normal Human 21 secs ago
0
Q: How to find a point in a Right Triangle given 2 known points, all sides, all interior angles

Adan RamirezThis triangle is not parallel or vertical, it's in a 2d plane. Problem Triangle, missing point

 
3:59 AM
0
Q: How do we decide a problem is in NP, but not in P or NPC?

Gaurav AradhyeAs I understand, NPC set contains only the problems which can be polynomially converted into each other and which are hardest in NP set/ But how do we decide which problems are in NPC and which problems are not in NPC but only in NP (not even in P)? Or there are no such problems which are in NP,...

Questions tend to get more attention when they have a tag for a broad area of mathematics relevant to the question. Some of these tags might fit. (autocomment)Normal Human 21 secs ago
 
0
Q: Reviving old questions or asking a new one?

swengerI'm relatively new to the Chemistry SE community, and sometimes when I go to ask a question, I notice that the same (or similar) question has already been asked. However, the previous question doesn't have a good answer (or is unanswered). In this case, is it better to ask the question again in a...

 
0
Q: Question about Multilinear Forms

AnfängerLet $\Delta$ be a skew-symmetric n-linear map. I have the following in my notes and I am having trouble seeing how it follows: $$ \Delta(\sum_{i=1}^n{e_i}, \sum_{i=1}^n{(e_i)} -e_2, \sum_{i=1}^n{(e_i)} -e_3, ..., \sum_{i=1}^n{(e_i)} -e_n) = \Delta(\sum_{i=1}^n{e_i}, -e_2, -e_3, ..., -e_n) $$ I ...

Words such as question do not add information to titles. Please edit the title so that it better describes the specifics of your question. Do not hesitate to make it longer or include a formula if needed. More tips here. (from a bot)Normal Human 21 secs ago
0
Q: Poisson process, exponential arrivals problem

nettleI cannot for the life of me see why my solution method gives me the wrong result. A machine works for an exponentially distributed time with rate $\mu$ then fails. A crew checks the machine at times according to a Poisson process with rate $\lambda$; if the machine has failed, it's immediatel...

0
Q: CDF on unit square

jazzinsilhouette A point $(a,b) \in [0,1]^2$ is selected at random. Let $X: [0,1]^2 \to [0,2]$ be the random variable that maps every point to the sum of its components, i.e. $X(a,b) = a + b$. Find the CDF $F(x) = \mathbb{P}(a + b \leq x)$ by considering the cases $0 \leq x \leq 1$, $1 < x \leq 2$ and $2 < x$ ...

Short title. CDF on unit square
 
I found the normal human!
:D
 
0
Q: among 2k integers there are two of equal parity differing by 2k-1 or two of opposite parity differing by 4k-2

CharlesProve that every set of $2k$ integers has elements of opposite parity differing by at least $2k-1$ or elements of equal parity differing by at least $4k-2$

 
4:23 AM
0
Q: Global convergence versus convergence to a global

RustyStatisticianI have been reading many optimization papers and wanted to know what the difference between global convergence and convergence to a global is. Sounds like the same thing to me.

 
4:38 AM
0
Q: Editing existing answers by adding additional information

AzkerMI'm pretty new to AskUbuntu's review section. IMHO, I review by doing some search before taking any decisions. But, hey! that is me. My question is, do we allow editing averagely voted answers with adding few more information? e.g.: What is the linux-image-extra package for and do I need it? ...

 
0
Q: Combinatorics, probability exercise

soullessI have a couple of combinatorics and probability problems that I have so much uncertainty when solving. My concepts are not that clear. Please shed me some light 1a You throw two dice. Whats the probability that the sum is 8 and none of the dice shows a 1. my attempt: 5/36 b. You have a coin y...

This site uses MathJax formatting of formulas. More tips here. (autocomment)Normal Human 21 secs ago
0
Q: Cheeger-Gromoll splitting proof

Enigma This is a proof from Eschenburg and Heintze's paper, An Elementary Proof of the Cheeger-Gromoll Splitting theorem. So I don't understand how the author gets the last three lines of the proof. I don't see what it means to take the norm of the hessian since its a $(0,2)$ tensor. Furthermore, I do...

0
Q: Stationary accumulation points

RustyStatisticianI have been reading about trying to prove global convergence of general optimization alrgoithms and am have come across the term "stationary accumulation point" and am trying to decipher exactly what that means. I read the definition of an accumulation point on Wikipedia (also called a limit poi...

0
Q: Irreducible Problem on Zp

EklavyaShow that $x^4+1$ is irreducible over $Z_p$ for every prime $p$.i have done it for $p=2,3$.but no idea as to how to do for general $p$??

Short title. Title contains problem. Short question. Irreducible Problem on Zp
 
1
Q: Specific question page ask me for username and password

Masih AkbariToday i come across something very odd a browsing throw site i saw a specific question page asked me for username and password!! what is it? is this a bug? please take a look your self, it happens with every browser and not dependent to if you are logged in or not.

 
0
Q: Key lemma for Akizuki (noncommutative case)

VdmLet R be a ring (not necessarily commutative) and maximal ideals M1,...Mn such that M1*...*Mn=0 then R is noetherian ring if and only if R is artinian ring

0
Q: Probability at least one event ocurring given correlations

MatthewI have a problem which I can't solve. I have three variables, lets call them A, B, and C, and their specific probabilities of them ocurring, P(A), P(B), P(C). I also have the correlation matrix between them, I'm using the Pearson correlation coefficient calculated using matlab. These three events...

This site uses MathJax formatting of formulas. More tips here. (from a bot)Normal Human 21 secs ago
0
Q: If $f(x)\geq 0$, is continuous in the interval $[0,\infty)$ , and $\lim_{x\to \infty} f(x)=0$, then $\int_1^{\infty} f(x)dx$ is finite.

faizuIf $f(x)\geq 0$, is continuous in the interval $[0,\infty)$ , and $\lim_{x\to \infty} f(x)=0$, then $\int_1^{\infty} f(x)dx$ is finite. This is true or false. Please someone help me.

 
5:03 AM
0
Q: How to prove that $(\frac{1+\tanh x}{1-\tanh x})^3=\cosh 6x+\sinh 6x$

Vinay5forPrimeHow to prove that $$(\frac{1+\tanh x}{1-\tanh x})^3=\cosh 6x+\sinh 6x$$ I have tried using the Dmoivres theorme

0
Q: If f bounded an left-continuous, can f be nowhere continuous?

NieblaIf a function f is bounded on [a,b] and the left-hand limit exists at each point of (a,b], can the function be nowhere continuous on [a,b]?

0
Q: Root of an equation given by an infinite series

SelfstudierConsider the equation $$e^{-x} = x-1 $$. We know that there is only one real root, $r$. How can it be shown that $$r= 1 + \sum_{n=1}^{\infty}\frac{(-n)^{n-1}e^{-n}}{n!}$$

0
Q: If $P \in Syl_p(G)$ and $P$ is cyclic, then $N_G(P)=C_G(P)$

Henri LLet $G$ be a group such that $|G|=p^a m$ where $p$ is the smallest prime divisor of $|G|$. If $P \in Syl_p(G)$ and $P$ is cyclic, then $N_G(P)=C_G(P)$ Proof First, note that $C_G(P) \leq N_G(P) \leq G$. Thus, we are done if $|N_G(P)|/|C_G(P)|=1$. Since $P \leq G$, by Corollary 1...

0
Q: A conceptual doubt in group theory

EklavyaAs we all know that in group $S_n$ every pair of disjoint cycles commute .my doubt is is it reverse all true,mean if a pair of cycles commute ,then they have to be disjoint??.i tried to find examples where cycles commute but not disjoint,but fail to do so

Words such as doubt do not add information to titles. Please edit the title so that it better describes the specifics of your question. Do not hesitate to make it longer or include a formula if needed. More tips here. (autocomment)Normal Human 21 secs ago
 
5:24 AM
0
Q: Correlation coefficient of a block matrix

tehseen fatimaMy question is related to the computation of correlation coefficient of a block covariance matrix. The correlation coefficient can be computed as: r = $ cov(X,Y)/std(X) std(Y) $ But if I have a block covariance matrix $A=[Cxx $ $ Cxy $;$Cyx $ $ Cyy $] (where C is $R^2$ and A is $R^4$)and I wa...

Consider adding a tag for a broader subject area to which the question belongs. Some of these tags might fit. (from a bot)Normal Human 21 secs ago
0
Q: Combinatorics partitioning problem: find $\sum_{n\geq 0} a_{n,k}\frac{x^n}{n!}$

pretzelman'If $a_{k,n}:=$ the number of ways of partitioning $n$ distinct objects into $k$ odd parts, what is $F_k(x)=\sum_{n\geq 0} a_{n,k}\frac{x^n}{n!}=?$' If I understand correctly, $a_{k,n}$ is the $n$th coefficient of the generating function $\left(\frac{e^{x}-e^{-x}}{2}\right)^k=F_k(x)$. This seem...

0
Q: 70 distinct positive integers that are ≤ 200, there must be two whose difference is one of 4, 5, or 9

Michael ColfaxProve that among 70 distinct positive integers that are ≤ 200, there must be two whose difference is one of 4, 5, or 9. So from this there are 582 possible pairs whose difference is 4,5, or 9. (i.e. {5,1},{6,2},...,{200,196};{6,1},{7,2},...,{200,195};{10,1},{11,2},...{200,191}). Thus, since th...

Welcome to Math.SE, Michael Colfax. Consider adding a tag for a broader subject area to which the question belongs. Some of these tags might fit. (autocomment)Normal Human 21 secs ago
0
Q: Simple power series convergence question

WaddleSo for my particular power series, I find that my interval of convergence is -3 \lt x \lt 3 so R \eq 3 I do a nth term test on the original equation with -3 and 3 and find that the series converges on these values. Does that mean the series is absolutely convergent at -3 \lt x \lt 3 and condi...

0
Q: Proof by induction -inequality

misheekohProve that $ 1 + \frac{1}{2}+ \frac{1}{3} + .... + 1/n < 2\sqrt{n}$ for $n \ge1$ Here's my attempt: Base case: $P(1): \frac{1}{1} \le 2\sqrt{1} = 1 \le 2$ (Base case true) Assume $n = k$ for $k \ge1$ such that $ 1 + \frac{1}{2}+ \frac{1}{3} + .... + \frac{1}{k} < 2\sqrt{k}$ INDUCTION HYPOTHE...

Tagged proof-verification. Proof by induction -inequality
Welcome to Math.SE, Waddle. Words such as question are uninformative in titles. Please edit the title so that it better describes the specifics of your question. Do not hesitate to make it longer or include a formula if needed. More tips here. (autocomment)Normal Human 27 secs ago
0
Q: To find sharp infimum (lower bound) of function with indicator function

userLet $(x_\varepsilon,y_\varepsilon)\in[0,1]\times[0,x_\varepsilon)$ be a sequence such that $(x_\varepsilon,y_\varepsilon)\to(x,y)\in[0,1]\times[0,x)$ as $\varepsilon\to0$. Is there an integrable function $f:[0,1]\to\overline{\mathbb{R}}$ such that $$ -\inf_{\varepsilon>0}\frac{\chi_{[0,y_\v...

Consider adding a tag for a broader subject area to which the question belongs. Some of these tags might fit. (autocomment)Normal Human 21 secs ago
 
5:39 AM
-1
Q: Rewarding people for giving amazing answers? (beyond upvoting)

silentphoenixToday, I was exploring the featured section of SO and I happened upon this question: Demystifying sharedctypes performance They question itself was fascinating, but look at the answer! Its like a mile long! I read it all and I think I learned a thousand things just reading it. It was amazingly ...

 
0
Q: given n stairs, how many number of ways can you climb either step up one stair or hop up two?

guest11this is the question given n stairs, how many number of ways can you climb either step up one stair or hop up two? I need to include the number of ways for n=1 through 6 as well. My question : is step up two stairs and hop up two, is it the same thing? however, I tried to do the solut...

This site uses MathJax formatting of formulas. More tips here. (autocomment)Normal Human 21 secs ago
0
Q: Show that $P^2$ = $P$

ChrisLet $P$ = $A(A^TA)^{-1}A^T$, where A is $m x n $ 0f rank $n$. This is the projection matrix, right? Every site I've been on says that this is the projection matrix such that $P^2$ = $P$, but none explain why. Is this just a property of a projection matrix that doesn't require proof?

0
Q: Proof for **u · v** = (**u · w<sub>1</sub>**)(**v ·w<sub>1</sub>**) + ... + (**u · wn<sub>n</sub>**)(**v · w<sub>n</sub>**)

DaneBrickSuppose {w1, w2, ... wn} was an orthonormal basis for Rn and u and v were vectors in Rn. I'm trying to prove that u · v = (u · w1)(v ·w1) + ... + (u · wnn)(v · wn) I know that since {w1, w2, ... wn} is an orthonormal basis that spans a subspace that contains u and v, u can be rewritten as (u ·...

This site uses MathJax formatting of formulas. More tips here. (from a bot)Normal Human 21 secs ago
0
Q: Consider the relation R on Z as: ∀m,n ∈Z, mRn ⇔ m − n is odd . Is R reflexive, symmetric, or transitive?

Todd BenjaminProvide a complete proof or counterexample for each property. And I have no idea what I'm doing.

0
Q: Confused about this exercise question: if a set with a certain binary operation is a group

a studentI tried to answer the following exercise: Let $S$ be a nonempty set with an associative operation that is left and right cancellative. Assume that for every $a$ in $S$ the set $\{a^n \mid n=1,2,3, \dots \} $ is finite. Must $S$ be a group? My thoughts: Take the group $G = \mathbb Z / 5 \mathbb...

Title contains question, confus, exercise. Tagged proof-verification. Confused about this exercise question: if a set with a certain binary operation is a group
0
Q: Why does $\sum_{J\subset I\subset S/F}(-1)^{|I|}=0$? (Alvis-Curtis Duality)

Denise GiSuppose $S$ is the reflection generators of the Coxeter group of some reductive algebraic group $G$. Let $F$ denote the Frobenius automorphism. The Alvis-Curtis duality $D_G$ is known to an involution on virtual characters, and part of the proof boils down to $\sum_{J\subset I\subset S/F}(-1)...

Questions tend to get more attention when they have a tag for a broad area of mathematics relevant to the question. Some of these tags might fit. (autocomment)Normal Human 21 secs ago
0
Q: $\mathbb{Z}^{\oplus \mathbb{N}} \times \mathbb{Z}^{\oplus \mathbb{N}} \cong \mathbb{Z}^{\oplus \mathbb{N}}$

Kelvin SohI am trying to prove $\mathbb{Z}^{\oplus \mathbb{N}} \times \mathbb{Z}^{\oplus \mathbb{N}} \cong \mathbb{Z}^{\oplus \mathbb{N}}$ and will appreciate hints to approach the question. Background: This question is taken from Aluffi. $\mathbb{Z}^{\oplus \mathbb{N}}$ is defined as $\{ \alpha \col...

A title should not be all-MathJax; having some plain text helps with search and navigation. (autocomment)Normal Human 21 secs ago
 
6:12 AM
0
Q: Find the general formula for the sequences

guest11 1=1 2+3+4+=1+8 5+6+7+8+9=8+27 10+11+12+13+14+15+16=27+64 Find the formula is suggested by these equations?Prove your answer is correct. I saw this question on practice exam and the answer is = (2n-1)(n²-n+1) but I only know a few steps and didn't get this answer. Please expl...

This site uses MathJax formatting of formulas. More tips here. (autocomment)Normal Human 21 secs ago
0
Q: Prove the inequality for x>y>0: xln(x) + yln(y) >(x+y)ln((x+y)/2)

DodgeI tried to solve it by differentiate both side and find the first derivatives are the same, then to use Rolle's theorem twice. However,I cannot continue from that.

0
Q: Wheel factorial: getting the next multiple

Tejas HsWhen doing wheel factorial by excluding 2, 3 and 5 I will have an equation of 30K+C=y to get the list of possible primes where C belongs to {1,7,11,13,17,19,23,29} and K belongs to {1,2,...} now I start with 7 and strike off its composites, the next composite of 7 is 49 (30*1+19), the others...

This site uses MathJax formatting of formulas. More tips here. (from a bot)Normal Human 21 secs ago
0
Q: How do I expand \int e^{x^2} \, dx?

user294826I know this cannot be finitely expanded but how can I expand it using a Taylor or a Power series? \int e^{x^2} \, dx

 
6:34 AM
0
Q: Are User allowed to promote their YoutubeChannel?

AltoyrI came across this question in the review queue. The user gives a vaque answer (I am not really familiar with this topic) and afterwards promotes a playlist in his YoutubeChannel. The playlist is about Hadoop as also is the question. It also is rather lengthy and seems to be of ok-quality. Is ...

 
0
Q: conservation law for the trajectories

AlanIs it possible to find the conservation equation as the form of $Q=h(x,y)$, given that $$\dot{x}=x-xy$$ $$\dot{y}=5xy-5y$$ I am not sure how to start with.

0
Q: extrema problem real analysis

DodgeIs it true that global maximum of a convex function f on [a,b] must be attained at the boundary? prove it or provide a counterexample. Is it possible for f to have a local maximum in (a,b) ? I know the answer intuitively, but I am not quiet sure how to prove it formally.

Short title. Title contains problem. extrema problem real analysis
0
Q: Show that $P$ is symmetric.

ChrisLet $P$ = $A(A^TA)^{-1}A^T$, where A is an m x n matrix with rank $n$. I feel like this is wrong, but here is my attempt: $A(A^TA)^{-1}A^T$ = $AA^{-1}(A^T)^{-1}A^T$ = $I$ And $I^T$ = $I$, so the matrix is symmetric.

0
Q: How to estimate the maximum likelihood

AbhinavQuestion: Based on the random sample $Y_1 = 6.3$ , $Y_2 = 1.8$, $Y_3 = 14.2$, and $Y_4 = 7.6$, use the method of maximum likelihood to estimate the parameter $\theta$ in the uniform pdf $f_Y(y;\theta) = \frac{1}{\theta}$ , $0 \leq y \leq \theta$ My attempt: L($\theta$) = $\theta^{-n} $ So, t...

Consider adding a tag for a broader subject area to which the question belongs. Some of these tags might fit. (autocomment)Normal Human 21 secs ago
 
6:52 AM
0
Q: OpenVpn using port 443 with Plesk

user294206I'm having a lot of trouble trying to use OpenVpn from the port 443 I'm using Plesk, and it's giving me a hard time trying to configure stuff in it. I went to this website to try to make Ovpn listen on port 443, but it didn't work and as I tried too much fixing it, my website doesn't show up in ...

 
0
Q: Show $\int_{\gamma}fg'dz=f(\beta)g(\beta)-f(\alpha)g(\alpha)-\int_{\gamma}f'gdz$

Tony let $G$ be an open set in $\Bbb C$ and let $f,g:G\to \Bbb C$ be analytic functions. Show that if $\gamma :[a,b]\to G$ is a rectifiable path with $\gamma(a)=\alpha,\gamma(b)=\beta$ then $$\int_{\gamma}fg'dz=f(\beta)g(\beta)-f(\alpha)g(\alpha)-\int_{\gamma}f'gdz$$ This looks like the integra...

0
Q: The order of $ab$ when $a,b$ commute

a studentLet $a,b$ be two group elements of finite order that commute. I thought that $|ab| = \text{lcm}(|a|,|b|)$. My proof was that $(ab)^n = a^n b^n =e$ if and only if $a^n = b^n = e$ if and only if $|a|,|b|$ both divide $n$. The smallest $n$ such that both orders divide it is the least common multip...

Tagged proof-verification. The order of $ab$ when $a,b$ commute
0
Q: function being analytic

IllustionistSuppose $g(z) = \Sigma_{n = 1}^{\infty}\frac{log n)^p}{n^z}$, where p is an integer. I want to prove that this is analytic I proved before that the rienman zeta function is analytic as it uniformly converge. I thought somehow I could use this to prove that f converges.

0
Q: Is an analytic function equal to its Taylor series at endpoints?

Liddel LokkerIs a real analytic function equal to its Taylor series at the endpoints of the interval of convergence, provided the series converges a the endpoint?

 
7:10 AM
0
Q: What happens to good, well-served, but abandoned, questions?

TheBlastOneQTP VBScript type mismatch error in function without arguments What happens to good, well-served, but abandoned, questions like this? Currently, they keep lurking around for years, without an accepted answer, showing up in the unanswered questions list. This really spams if you are looking for g...

0
Q: Rules for connected tags or tags with prefix

oberflanschI just stumbled over the following question: Tag synonyms: android-avd and avd Question: I would be interested in: what should be preferably used in general and are there some rules regarding tag creation and usage? If there is a risk of confusion, wouldn't it be better to generally use androi...

 
0
Q: $e^{-\frac{1}{x}}e^{-\frac{1}{1-x}}$ in 3D

Константин ВысоцкийI have the function $f(x) = e^{-\frac{1}{x}}e^{-\frac{1}{1-x}}$, which produces this graphic. What should $f(x,y)$ be to look like a 'hill', i.e. $f(x)$ spinned about vertical axis?

Short title. Short question. $e^{-\frac{1}{x}}e^{-\frac{1}{1-x}}$ in 3D
0
Q: How to integrate $xe^x$

Angelo MarkYesterday I sat for my Real analysis II paper. There I found a question asking to integrate $\int_{0}^{1}xe^x$ without using antiderivatives and integrating by parts. I tried it by choosing a partition $P_n=(0,\frac{1}{n},\frac{2}{n},.......\frac{n-1}{n},1)$. But I was not able to show that $li...

0
Q: Write a notation for a vector

KimI would like to write a notation for a linearly spaced vector. This vector has 21-scaler or element. The max of vector is starting point and min of vector is stopping point. Min is max times 0.2, and the rest of elements were linearly spaced between max and min. May someone help me to write t...

Welcome to Math.SE, Kim. Tag (notation) should not be the only tag a question has. Please add a tag for a subject area to which the question belongs. (from a bot)Normal Human 21 secs ago
 
0
Q: What to do about Edition Take-over which adds nothing new to edit?

NSNoobFirst off, this is not the classic case of "My name is not showing as editor" as cited here and here. Two of my recent edits were recently taken over by users with 2k+ reputation. First one was this. When I proposed the edit, the user had everything jumbled up in code block. I rectified it. Later...

 
0
Q: Prove any polynomial of degree n that is orthogonal to ${1, x, ..., x^{n-1}}$ is a constant multiple of a Legendre Polynomial.

NimmerThe Legendre Polynomials are defined by $L_n(x) = \frac{d^n}{dx^{n}} (x^2 - 1)^n$. The inner product in this case is defined on $[-1, 1]$ as follows: $<f(x), g(x)> = \int_{-1}^{1} f(x)g(x)dx$. I'll denote the arbitrary polynomial of degree n by $P_n(x)$. Since it is orthogonal to $\{1, x, ..., ...

 
7:34 AM
0
Q: Integrate \int{ \frac{r}{(h^2 + r^2 - 2^{1/2}hr)^{1/2}} dr }

A.ZThis integral comes from a physics book when calculating potential difference between vertex and center of cone. I'm not good at integration. Please help.

Short question. Question contains please. Integrate \int{ \frac{r}{(h^2 + r^2 - 2^{1/2}hr)^{1/2}} dr }
0
Q: Calculating $Log(-e i)$

Raphael $$Log(-e i)$$ My try: $$=\ln|0+(-e)i|+i[\arg (0+(-e i))+2\pi k]$$ $$=\ln|e i|+i(-\frac{\pi}{2}+2\pi k)$$ My attempt is correct?

Short title. Tagged proof-verification. Calculating $Log(-e i)$
0
Q: If $A$ is a complex matrix satisfying $A^3=I$ then what are the possible (complex) eigenvalues of $A$?

probabilitydoesntsuckAs the title says, nothing about the dimensions of the matrix is mentioned? What should I be starting with here? I just know $$|A-\lambda I| = 0 \\ and \\ A^3=I$$

 
8:06 AM
0
Q: Gaussian Copula Function

MatthewI have a question that I believe I know the answer to, but I would like to double check. Is it possible to use a copula function to calculate the joint probability of three or more variables? I'm asking because on this PDF https://www.math.ust.hk/~maykwok/courses/Dyn_Cred_Models/Topic2.pdf on ...

 
8:21 AM
0
Q: Undergraduate vs Graduate Real Analysis

playitrightAt my institution, we have what is called "Introduction to Modern Analysis I" and "Introduction to Analysis II" i.e. a year long sequence in real analysis. It's often branded as the "killer course(s)" of the math major, yet it's labeled as an introductory course. Is real analysis so involved that...

Tag (soft-question) should not be the only tag a question has. Please add a tag for a subject area to which the question belongs. (autocomment)Normal Human 20 secs ago
0
Q: moving pipe around corner

martinI worked at some problem with moving pipe around a corner and i was trying to fing the maximum length of pipe for corner with corridor lengths 1. It is pretty obvious that maximum length is {\sqrt 2}, since that is the lenght of pipe when it's in position, when the angles where it touches the out...

 
8:34 AM
0
Q: Proving that a homomorphism is abelian

Stephanie HernandezThe question states... Let phi: G to G be a homomorphism with the map phi(g)=g^2. Prove phi is abelian. So far I have: Let g and h be in G. Then phi(g)=g^2 and phi(h)=h^2. Since we know it is a homomorphism, Phi(gh)=phi(g)*phi(h)=g^2*h^2.... But I don't know how to make a connection that they...

This site uses MathJax formatting of formulas. More tips here. (autocomment)Normal Human 20 secs ago
0
Q: integration with e^cos(t)

user2615936I am trying to define the steps for the integration of the following $$\int\limits_{0}^{\frac{\pi}{2}}\left((cos(t))(cos^2(t)+2sin(t)e^{cos(t)}+1)+(-sin(t))(2sin(t)cos(t)+sin^2(t)e^{cos(t)}+2cos(t))\right)dt$$ I am plugging this into wolfram alpha and it shows that the answer is 1, but I have no...

 
8:51 AM
0
Q: Unbiased estimator problem

AbhinavLet $X_1, X_2, . . . , X_n$ be a sample of size n from a distribution with unknown mean $−∞ < µ < ∞$, and unknown variance $σ^2 > 0$. Show that the Y = $(X_1 + 2X_2 + 3X_3 + ... + nX_n) / (1 + 2 + 3 .... n )$ is an unbiased estimator of µ

Short title. Title contains problem. Unbiased estimator problem
0
Q: How to generate the next permutation?

user3002228I feel confuse with the next permutation. We have 4 step for find the next permutation. step 1: find from right to left and check a[i] < a[i+1]. step 2: find the pivot with condition is a[k] < a[i]. step 3: swap a[i] vs a[k]. step 4: sort list with reverse. how to generate the next permutation o...

This site uses MathJax formatting of formulas. More tips here. (autocomment)Normal Human 21 secs ago
0
Q: find an example of a homomorphism of D4 to Z8 such that ker={e,R,F}

Stephanie HernandezThe question asks to find an example of a homomorphism of D4 to Z8 such that ker={e,R,F}. D4 = {e, R,R^2,R^3,F,FR,FR^2,FR^3} and Z8={0,...,7} I thought about letting the map equal: Phi(e)=0 Phi(F)=0 Phi(R)=0 But I wouldn't know how to map the other elements... Phi(R^2)= Etc

This site uses MathJax formatting of formulas. More tips here. (autocomment)Normal Human 21 secs ago
0
Q: Approximating on a line

Eric S.Say I have sampled some points in $[0,1]^2$ and evaluate a function $f(x,y)$ for them. I am interested in the behavior of $f$ along a single dimension. If the points were like $(x_1,y_1),(x_1,y_2),\ldots,(x_1,y_N)$, then they lie nicely on a straight line, such that I can compare $f(x_1,y_1),f(...

 
9:17 AM
0
Q: $x$ varies as $y^2$.

Sufyan Naeem$$x \propto y^2$$ How is it different from saying: $$x \propto y$$ That is; when we say that Two variables are proportional then it means that two variables are related such that when one is zero other is too. And change in one variable is accompanied by change in other. This is a general defi...

Short title. $x$ varies as $y^2$.
0
Q: derivative and extrema problem

ashconsider f(x) = x^{2} sin(1/x) + x^{3} on R, asuume that f(0) = 0. (a) prove that f is differentiable on R. (b) Does f have a local extremum(minimum or maximum)at x = 0? explain for part a, I tried to use definition of derivative, but don't know how to compute derivative of f(x) correctly. f...

Title contains problem. derivative and extrema problem
0
Q: Approximating continuous function vanishing at infinity on product space by product of functions

user92360I am not sure if the question I am asking is proper enough: Is there any way to approximate any arbitrary function $f(x,y) \in C_{0}(\Omega $ X $ S)$ in the uniform norm by linear combinations of functions of the form $f(x)g(y)$ (which is in $C_{0}(\Omega)$ X $C(S)$)??? [Where: $C_{0}$ denote...

0
Q: HELP please. Improper Integral of e^-e^x from 1 to infinity

Karn PiriyapanyaI have no idea to start or consider the integral \int_{0}^{\infty} e^{-e^{x}}dx any hint or solution would be highlt appreciated. Thank you.

Welcome to Math.SE, Karn Piriyapanya. Words such as help, please are uninformative in titles. Please edit the title so that it better describes the specifics of your question. Do not hesitate to make it longer or include a formula if needed. More tips here. (autocomment)Normal Human 21 secs ago
0
Q: One point union, second homotopy group is not finitely generated?

user294892Let $X$ be the one-point union of $S^1$ and $S^2$. What is the easiest way to see that the abelian group $\pi_2(X)$ is not finitely generated?

0
Q: When do minimizers exist?

Denis DüsseldorfI'm trying to solve a problem set for my functional analysis course and I'm stuck at the following problem: Decide if the following problem has a minimizer Let $g\in C^0([0,1])$. Minimize $\|f-g\|_{L^\infty([0,1])}$ among all $f\in L^\infty([0,1])$ with $$\int_0^1\,f\,dx=0$$ Hint: C...

0
Q: normal cone inclusion and non-symmetric matrix and optimization problem

GabrielI have the following normal cone inclusion $$-(A x + b) \in \mathcal{N}_\mathcal{C}(x)$$ where $\mathcal{N}_\mathcal{C}$ denotes the normal cone to the convex set $\mathcal{C}$ at the point $x \in \mathcal{C}$. Matrix $A$ is non-symmetric. Normally if $A$ would be symmetric, the convex optimiz...

0
Q: How do I prove that among any 5 integers, you are able to find 3 such that their sum is divisible by 3?

Ilia LabkovskyI realize that this is a number theory question and we use modular arithmetic, but I'm unsure of where to begin with this specific situation.

0
Q: Measure of non-compactness

Shinning StarCan someone give me some simple examples of measure of non-compactness of sets in Banach spaces or metric spaces, which are easy to understand.

Short title. Short question. Measure of non-compactness
0
Q: Number of Distinct ideals of Z60?

SaurabhTried to count all prime numbers between 0 to 60 and adding (0) and (R) to it. that is total 19 , but I saw that answer is 18. So, Please explain.

Short question. Question contains please. Number of Distinct ideals of Z60?
0
Q: linear problem with $\|.\|_\infty$ and $\|.\|_1$ norm constraints

DaveI have a question regarding a straightforward linear algebra problem, yet the solution is (at least for me) not trivial. Assume the sequences $\phi_i$ with coefficients $\phi_i[n]\in\mathbb{R}$, and coefficients $b_i\in\mathbb{R}$ given by the problem. Now the objective is solving the followin...

0
Q: Number of group homomorphism from $Z_8$ ⊕ $Z_4$ to $Z_4$ ⊕ $Z_4$

mehul jainI know that there does not exist a onto homomorphism from $Z_8$ ⊕ $Z_4$ to $Z_4$ ⊕ $Z_4$ but what about into homomorphisms. How many of them exist and how to find them.

 
10:06 AM
0
Q: Inline tag edits do not directly update post signature

Tim CastelijnsWhen you edit a post with the edit button, the post is edited and the post signature is directly updated with the editor's signature: However, when using the inline tag edit feature (10k+ user feature): the post is edited but the post signature is only updated after a page reload. I believ...

 
0
Q: Variational formulation of elliptic mixed boundary value problem

avati91I have some troubles with variational formulations of pdes. I'm considering $\chi\in H^1(\Omega)$ is the solution to the elliptic mixed boundary value problem $$ \begin{cases} \nabla\cdot(\mu \nabla\chi)=\nabla\cdot(\mu\chi)\;\,\text{in}\,\Omega\\ \mu\nabla\chi\cdot n=\mu v\cdot n\;\,\text{on}\,\...

0
Q: convex function and local extrema problem

ashLet f be twice differentiable and strictly convex on [a,b]. Assume also that at a point x0 ∈ (a,b) the derivative f'(x0) = 0. Show that x0 must be a strict local minimum. I find that when xx0, f'>0. So if I can conclude that for all x in the interval, f(x) > f(x0) and therefore x0 must be a stri...

0
Q: Find the value of $f'(2)$ where $f(x)=\lim_{n\to\infty}\sum_{n=1}^{n}\arctan(\frac{x}{n(n+1)+x^2})$

Vinod Kumar PuniaFind the value of $f'(2)$ where $f(x)=\lim_{n\to\infty}\sum_{n=1}^{n}\arctan(\frac{x}{n(n+1)+x^2})$ I could not find $f(x)$ here.I had a feeling that Riemann integral should be used to find the $f(x)$ but i cannot find after some efforts.Please help me.

0
Q: Eccentricity and level-sets

SashaDefinition of eccentricity: Let $N$ be a surface. Consider $[R(N), r(N)]$ to be the smallest interval such that $N$ is contained in the annulus $D_R \setminus D_r$. We define the eccentricity as $\Theta(N) =R(N)/r(N)$. Now, I consider level-sets of a function $v$ in $\mathbb{R}^n \setminus \{...

 
10:24 AM
1
Q: Reputation not added for suggested edits

IrshadI have suggested some edits and 5 of them were approved. But reputation didn't added up. Same for yesterday. Is something wrong?

 
0
Q: How can we prove that there exists a $c$ such that

kileHow can we prove that there exists a $c$ such that: $$c\log(\log(n)) > \sum \cfrac{ 1}{p} $$ where p is a prime number $< n$ starting from $p = 2$

0
Q: Which bases are used in a number?

Zam123My question is asking me to find bases, 2 to 10 which are used in a number like 006284.how would you show your working.thanks

0
Q: Good book for learing algebra and sigma algebra?

Mayank TiwariI am beginner in probability theory. In order to make a better understanding of Borel Sets, Measurable Space and Random Variable, I need to learn about algebra and sigma algebra, Can anyone please suggest any good book that covers these topics for beginners with sufficient questions (questions on...

Question contains please, please. Good book for learing algebra and sigma algebra?
 
10:58 AM
0
Q: Birthday problem(probability)

soullessWhat is the probability that three people have birthday on the same day for a group of people n? My attempt: NC3 * (1/365)^3 * [364/365 * 363/364 .....(364-(N-3))/(365-(N-3))] Is this the way to solve it?

This site uses MathJax formatting of formulas. More tips here. (autocomment)Normal Human 21 secs ago
 
11:09 AM
0
Q: A broken clock's minute hand takes 59 minutes (instead of 60) to complete a cycle. Despite this, the hour hand still takes 12 hours for 1 cycle.

Ilia LabkovskyIf the hour and minute hand start at 12, how long until they both point to 12 again? With a normal clock, I know this would take 12 hours.

0
Q: Convergence of a improper integral

BalajiTest the convergence of the improper integral ` 1 to infinity 1/(x^2 .(1+ e^-x)) dx I have tried to change the fraction as (e^x)/(x^2 .(1+ e^x)) then it can be simplified as 1/x^2 - 1/(x^2 .(1+ e^x)) . Now I couldn't solve the integration of second term. Please help me

Welcome to Math.SE, Balaji. This site uses MathJax formatting of formulas. More tips here. (from a bot)Normal Human 21 secs ago
0
Q: KKT Condition for Optimality

user1234I have an objective function which is given as: $f=x_1+x_2+x_3+x_4+x_5+x_6+x_7+x_8$ S.T. $x_1+x_3+x_5+x_7=2$ $x_2+x_4+x_6+x_8=2$ $x_1(x_2)=0$ $x_3(x_2+x_4)=0$ $x_5(x_2+x_4+x_6)=0$ $x_7(x_2+x_4+x_6+x_8)=0$ Given the above constraints I want to know if and how this problem can be solved u...

0
Q: Find the exact sum of an infinite Series

anaPlease find the exact value of an infinite series $\sum r^{n(n+1)}$ for $0<r<1$ .

Short question. Question contains please. Find the exact sum of an infinite Series
0
Q: let v amd w be the subspaces of R^2 spanned by (1,1) and (2,2) find victors v amd w so v+w=(2,-1)

Shadalet v amd w be the subspaces of R^2 spanned by (1,1) and (2,2) find victors v amd w so v+w=(2,-1) i have this problem it's new idea for me and i don't know from where to start!

 
11:31 AM
0
Q: testing the convergence of a improper integral

BalajiFind all the real values of p and q so that the integral 0 to 1 x^p(log(1/x))^q dx converges. I tried using comparison test but couldn't solve it. Please help me.

Short question. Question contains please. testing the convergence of a improper integral
 
1
Q: Linking to a ticket fixing the question topic is wrong?

Jakub BochenskiI don't get it, a guy was pointing out some problem in Spring, I've filled a ticket for the exact same problem and got it accepted. So why was my answer linking to the ticket deleted/converted to a comment? I got This does not provide an answer to the question. To critique or request clar...

 
0
Q: show that the set V = {(x,y,z)|x,y,z in R and x+y=11} is not a subspace of R^3

Shadaa) show that the set V = {(x,y,z)|x,y,z in R and x+y=11} is not a subspace of R^3 b) let V = {(x,1/2x):x real number} with standard operations. is it a vector space justify your answer. c) let V = {(x,1/2x,x^2):x real number} with standard operations. is it a vector space justify your answer. ...

Welcome to Math.SE, Shada. This site uses MathJax formatting of formulas. More tips here. (autocomment)Normal Human 22 secs ago
0
Q: Prove convergence in $L^1$

Both HtobLet $(X, \mathscr{A}, \mu)$ be a finite measure space. Let $f_n \in L^1$. Assume $f_n \rightarrow f$ a.e. and there exist $p > 1$ and $c > 0$ such that $$||f_n||_p < c$$ for all $n$. I want to show that $$f_n \rightarrow f \ \mbox{in} \ L^1, \ \mbox{i.e.,}\ \int_X |f_n - f| d\mu.$$ Since $f_n \...

0
Q: Volume bounded by elliptic paraboloids

BalajiFind the volume bounded by the elliptic paraboloids given by z=X^2 + 9 y^2 and z= 18- X^2 - 9 y^2. First I found the intersection region, then I got x^2+ 9 y^2 =1. I think this will be area of integration now what will be the integrand. Please help me.

This site uses MathJax formatting of formulas. More tips here. (autocomment)Normal Human 26 secs ago
0
Q: Gamma function, showing identity of terms

TonioHow can I show this identity?: $$t^{a+h-1}e^{-t}=t^{a-1}e^{-t}\sum_{k = 0}^{\infty} \frac{ \log(t)^k}{k!}h^k $$ Anyone can give me hint?

0
Q: Proving that the circumcenters are concyclic.

anonymousI was completely lost when handed this at a math competition a couple of weeks ago. I drew the diagram and was able to make sense of the question. My diagram also seemed to show that the circumcenters were concyclic however, because of my lack of any major exposure to circle geometry I was not...

0
Q: Hölder type inequality

Mark_HoffmanI was reading a paper and I found stated without proof or reference the following inequality: $$\left(\sum_{i=1}^{m}\prod_{j=1}^{n}(\beta_{i}^j)^p\right)^{\frac{1}{p}}\leq \prod_{j=1}^{n}\left(\sum_{i=1}^{m}\left(\beta_{i}^{j}\right)^{p_j}\right)^{\frac{1}{p_j}}$$ where $\beta_i^j\geq 0$ and $\...

0
Q: Show that $f(U)$ is open when $U$ is open.

MSELet $f:\mathbb R\to\mathbb R$ a continuous function. Show that $f(U)$ is open when $U$ is open $\iff$ $f$ is a bijection.

 
12:25 PM
0
Q: Something wrong with Chat room?

M DRecently just got one chat room invite http://chat.stackoverflow.com/transcript/96672 when i go inside nothing in. I don't know what's wrong. It's just show me room info and one message and also i can't send message in chat room. what's wrong here?

 
0
Q: Slope of the tangent to the circle

Lev WingmanA circle has its center at (3,4). Find the slope of the tangent to the circle at (5,-2). The points can be drawn on the Cartesian plane.

0
Q: Bayesian approach to probability in plain English

MichalHow would you define Bayesian approach to probability in a concise down-to-earth way?

0
Q: I got stuck to solve a path integral problem.

JAEMTOShow that the path integral of f(x,y) along a path given in polar coordinates by r=r($\theta$), $\theta_1$ ≤ $\theta$ ≤ $\theta_2$, is $\int_{\theta_1}^{\theta_2}$ f(rcos$\theta$,rsin$\theta$)$\sqrt {r^2+(\frac{dr}{d\theta})^2 }$ d$\theta$ I thought x=rcos$\theta$, y=rsin$\theta$ So r($\theta$...

Words such as stuck do not add information to titles. Please edit the title so that it better describes the specifics of your question. Do not hesitate to make it longer or include a formula if needed. More tips here. (from a bot)Normal Human 21 secs ago
0
Q: Show that every bounded above sequence $x_{n}$ tends to $-\infty$ or there exists $\limsup_{n\to\infty} \ x_{n}$

Jelly BellyI am stuck on how to show that either $\limsup$ does exist or the sequence tends to $-\infty$ when the sequence is bounded above. Can anybody help me?

0
Q: Travelling wave is an equilibrium (time independent) solution

M. MeyerConsider the following PDE. $$ u_t=u_{xx}+f(u)-w,~~~~~w_t=\varepsilon(u-\gamma w),~~~~~~~(1) $$ where $f(u)=u(u-a)(1-u), 0<a<\frac{1}{2}, \varepsilon,\gamma >0, \varepsilon\ll 1,\gamma\ll 1$. A travelling wave for (1) is a solution that is a function of the single variable $\xi=x-ct$, i.e. $(u(\...

0
Q: Set Theory: Proving sets

Lev WingmanProve: n(A∪B∪C)=n(A)+n(B)+n(C)-n(A∩B)-n(A∩C)-n(B∩C)+n(A∩B∩C) Using the laws of the algebra sets, prove that n(A∪B∪C) is equal to n(A)+n(B)+n(C)-n(A∩B)-n(A∩C)-n(B∩C)+n(A∩B∩C)

 
12:57 PM
0
Q: Should not we increase the minimum reputation requirement to review suggested edit?

manetsusI often fall in such types of embarrassing situations. It thought to write it before, but did not write for my laziness. However, all of you know that, a new user, with reputation point 1, can review suggested edits of his own question. But most of these new users do not know that when a sugges...

 
This site uses MathJax formatting of formulas. More tips here. (from a bot)Normal Human 21 secs ago
0
Q: How do I explain Konigsberg Bridge problem to a child?

tatanThis question may seem a little off-topic for this site. I am going to demonstrate Konigsberg $7$ bridges problem in a science exhibition.I am also going to use a model for a more visual representation of the problem.Now,how do I explain this (the solution) simply to a child who is not too mu...

0
Q: Using Fourier Transforms

JimmI'm new to Fourier Transforms and need some help using them to solve equations. Can someone explain to me how I could use a Fourier Transform to solve an equation like $3u_x + 5u_t = 0$ with $u(x, 0) = f(x)$ ?

Short title. Tagged pde, differential-equations. Using Fourier Transforms
0
Q: Having PI fingers and count

Per ErikssonThe number PI has infinite decimals whom appear to be randomly distributed. If we had PI fingers, and would therefore use PI as base instead of ten, could there exist other integers?

Short title. Short question. Having PI fingers and count
 
0
Q: Should we do something with [wlln] tag?

TimI just noticed that there is wlln. Could you guess what does it stand for..? It has only 11 questions and the topic is pretty broad. Its name is hard to search if you do not know that it exists, so it is probably useless. Maybe it should be renamed or merged with other tag that has more informati...

 
1:23 PM
0
Q: $\lim\limits_{n\ to \infty} {\frac{(2n-1)!!}{(2n)!!}}$

user1640736I have tried using Stolz-Ceszaro's formula and substract the next term in the series but that gave me $$\lim_{n\to \infty} {\frac{2n}{2n+1}}$$ which is obviously 1 and not right. I do realise that the limit should be 0 but I don't know how to prove it.

A title should not be all-MathJax; having some plain text helps with search and navigation. (from a bot)Normal Human 21 secs ago
0
Q: Linear Algebra- Find the matrix that represents the composite Linear transformation

Jon RoyI am stuck on the following question, Find the matrix that represents the composite function GTS where S(x,y,z) = (2x,y+z), T(x,y) = (2x + y, x + y, x-y), G(x,y,z) = (y,x,x,z)

0
Q: Integral $\int_{-\frac12}^3 \frac{\ln (x+2)^{\frac1{x+1}}}{\ln(x+3)+\ln(x+2)-\ln(x+1)}$

Kay K.Evaluate the following integral $$\int_{-\frac12}^3 \frac{\ln (x+2)^{\frac1{x+1}}}{\ln(x+3)+\ln(x+2)-\ln(x+1)}$$

0
Q: How to simplify an expression?

StudentI have tried to simplify this expression for quite a long time now but I can't find how to do it. Can someone help me with it.

 
0
Q: Show # of team members on a user's profile

AnzeoCurrently, you can see what teams a user belongs to, but IMO it would be a nice indicator to see the number of members (between brackets) in those teams. Example: Teams (3) Stack Overflow (55) Stock Everflaw (13) Stork Babyboom (5K)

 
1:53 PM
0
Q: Finding sequences that converges

GuestTest the following sequences for convergence. If a sequence converges, give its limit. Justify your reasoning. enter image description here

0
Q: Recurrent random walk

sky90Let $S_n=S_0+\sum_{i=0}^n{X_i}$ be a random walk with increment distribution $p$ and n-th step distribution $p_n(x)=\mathbb{P}[S_n=x\mid S_0=0]$. We say that a random walk is recurrent if $\mathbb{P}[S_n=0\ i.o.]=1$. Assume that we are in dimension 1, then we want to check that $S_n$ is recurren...

0
Q: Analytic Semigroup Norm Bound

User112358I am reading up on analytic semigroups from Renardy's An Introduction to Partial Differential Equations: http://uxmym1.iimas.unam.mx/ramon/docs/RenRog.pdf I am trying to prove Lemma 12.32: Let A be the infinitesimal generator of an analytic semigroup. Then there are constants $C$ and $\omega...

 
2:10 PM
-1
Q: Code formatting in the title

sawaCode fragments often appear in the title, and it is not pleasing to see it mixed within ordinary words, and it also takes a bit more time than regular words to recognize it. I would like to have the title allow inline code formatting with backtick escapes, so that wherever the title appears on ...

 
2:21 PM
0
Q: Interior and exterior

user294936X=<0,4]U{6}U[10,11] in R. Need to find interior and exterior of A=(0,2]U{6}U(10,11] in X? Any help would be very welcome! Thanks in advance.

Short title. Short question. Interior and exterior
0
Q: How to calculate these residues?

Yves Paschoal$$Res\left( \frac{e^z}{tan(z)}, \frac{\pi}{2} \right)$$ and $$Res\left( \frac{1}{Log^2(z)}, 1 \right)$$ Grateful if someone can show me a way to calculate these residues.

0
Q: Problem on application of Fubini's theorem?

uptowndowndownI have the following problem that I am having trouble with: Let $f$, $g$ be positive measurable functions. Let $$F(z) = \int_{\{g \geq z\}} f d\mu$$ Prove $$\int_X fg d\mu =\int_0^\infty F(z) dz$$ This does seems like I need to use Fubini's in some way, but I am a bit confused about $F(z)$ ...

0
Q: Prove that $a>b$.

AmartyaLet $F\subseteq \mathbb C$ be the splitting field of $x^7-2$ over $\mathbb Q$ and $z=e^{\frac{2\pi i}{7}}$ a primitive seventh root of unity. Let $[F:\mathbb Q(z)]=a$ and $[F:\mathbb Q(2^{\frac{1}{7}})]=b$ .Then prove that $a>b$. Now $F$ is the smallest field such that $x^7-2$ splits over $\mat...

Short title. Prove that $a>b$.
0
Q: Get rid of division in modulo

Ali_runI'm writing a program, that draws horizontal stripes and blank spaces between them with the same height 44. So I made an equation $({{y}\over{44}}) \mod {2} < {1} $ The problem is I'm not allowed to use the division in this programming language. Could you, please, help me to get rid of this d...

Short title. Question contains please. Get rid of division in modulo
0
Q: Quadratic Equation Application

user256670Textbook exercise I get stuck "Any is going to hold a concert at5 a stadium. The stadium can accomodate 12 000 poeple. If the price for each ticket is $160, all th etickets will be sold. For every increase of $1 in the ticketprice, the number4 of tickets sold will decrease by 50. Let $p be t...

Short title. Question contains please. Quadratic Equation Application
0
Q: Gradient of product of functions

zshaI have to find $$\nabla_c (c'Ac - 1)(Ac)$$ where $A$ is a symmetric matrix and $c$ is a vector. Can anyone give me suggestion on how to solve this type of gradient problem?

0
Q: Infinite series $\frac1{1+\ln x}+\frac2{1+(\ln x)^2}+\frac4{1+(\ln x)^4}+\frac8{1+(\ln x)^8}+\cdots$

Kay K.Evaluate the following expression $(x>e)$ $$\frac1{1+\ln x}+\frac2{1+(\ln x)^2}+\frac4{1+(\ln x)^4}+\frac8{1+(\ln x)^8}+\cdots$$

0
Q: Prove an inequality using mathematical induction

Gizmo of ArabiaHow do I prove that n!<(n/2)^n for n>=6, using mathematical induction?

Short question. Tagged proof-theory. Prove an inequality using mathematical induction
 
2:54 PM
-2
Q: How to challenge closing question as a duplicate by a single person?

AdamSkywalkerI've read this question on similar topic, but it's about closing a question by 5 people. I do not know what to do when question is marked as a duplicate by a person with gold badge. I have a gold badge too and I do not agree, can I protest? Five man can be enough for objective judgement but a si...

 
0
Q: Prove P is a vector space.

RossWe have a population $p_j$ in year $j$ is governed by the following equation: \begin{equation} p_{j+1} = 2p_j + 3p_{j-1} + p_{j-2} \end{equation} I want to prove that the set of all sequences, p = $(p_0,p_1,p_2,...)$ for $j \geqslant 2$ is a vector space, denoted by V. I started by taking 2 ve...

 
0
Q: This question was closed-as-dup but the accepted answer is out-of-date

joeytwiddleGoogle brought me to this question: Is there a way to identify the busy (CPU-consuming) tab in Firefox? But that question is marked as a duplicate of this one: How to find out which firefox tab is using most CPU or memory? The problem is that the second question already has an accepted answer...

 
0
Q: M is finitely generated but $ker(f)\subset M$ is not.

Carlos Vázquez MonzónSo I'm searching for an homomorphism of $A$-modules $f:M\mapsto N $ and some A-modules $M$, $N$ such both are finitely generated but Ker $f\subset M$ is not. Thanks.

0
Q: integrate given time frame

David ScidmoreI am give the equation Dew/dt = -1400(t-x)+350 where x I have tried Dew/-1400 = t-x -.25 Dew/-1400 + x = (t-.25)dt Dew -1400x = (t-.25)-1400dt Not sure where to go from here and think it is wrong as well.

0
Q: Calculation of Fourier series

ArupLet us define function $V(t) = \begin{cases}3, \text{for $0\leq t < 6$}\\ 4, \text{for $6\leq t < 12$} \\ 3, \text{for $12\leq t < 18$} \\ 0, \text{Otherwise}. \end{cases}$ What is the Fourier Series for $V?$ Is the usual formula i.e $a_n = \frac{1}{2\pi} \int _{-\pi}^{\pi}V(t)cos(nt)dt$ and $b...

Consider replacing (analysis) with a more specific tag for the relevant branch of analysis. (autocomment)Normal Human 20 secs ago
0
Q: Zero divided by zero must be equal to zero

LennartWhat is wrong with the following argument (if you don't involve ring theory)? Proposition: 0/0 = 0 Proof: Suppose that 0/0 is not equal to 0 0/0 is not equal to 0 => 0/0 = x , some x not equal to 0 => 2(0/0) = 2x => (2*0)/0 = 2x => 0/0 = 2x => x = 2x => x = 0 =>{because x is not equal to 0}=> ...

Welcome to Math.SE, Lennart. This site uses MathJax formatting of formulas. Tag (proof-verification) should not be the only tag a question has. Please add a tag for a subject area to which the question belongs. More tips here. (autocomment)Normal Human 20 secs ago
0
Q: Show that $f(z)$ is a polynomial of degree 1

Von KarI'm learning about complex analysis and need some help with this problem : Let $\Omega$ open connected and $f : \Omega \rightarrow \mathbb{C}$ analytic such that $f(z) \neq z$ for all $z \in \Omega$. Suppose there exists $a \in \Omega$ such that $\left| f(a) - a \right| \leq \left| f(z) - z \...

 
3:37 PM
0
Q: Function bounded from belowf

ZimkovicI want to show that $$g(f):=\int_{\mathbb{R}^3} |f'(t)|^2+9x^4(x^2+2)|f(t)|^2 dt$$ is bounded from below by $3||f||_{L^2}$ for $f \in C_c^{\infty}(\mathbb{R}).$ What is obvious is that $g$ is bounded below by $0,$ but I don't see how the $3$ comes into the game. Does anybody have an idea?

0
Q: Is $\sum_{n=0}^\infty nP(|X|\ge n) \ge E(|X|^2)$?

MathStudentThe question is in the title. I am doing a problem and wanted to know if this inequality holds: $\sum_{n=0}^\infty nP(|X|\ge n) \ge E(|X|^2)$.

0
Q: Figure 1.4 of Introduction to Calculus and analysis vol 1

potter johnFigure 1.4 I get stuck in here. Should it be 10 11 100 101 110? Thanks for your help!

Tag (reference-request) should not be the only tag a question has. Please add a tag for a subject area to which the question belongs. (autocomment)Normal Human 20 secs ago
0
Q: What are some necessary and sufficient conditions for "a rational function being uniformly continuous on $\mathbb{R}$?

Tien-Cheng HuangLet $f(x)=\frac{q(x)}{p(x)}$ for some polynomials $q(x)$, $p(x)$ with $p(x)$ nonzero on $\mathbb{R}$. What are some necessary and sufficient conditions for "$f(x)=\frac{q(x)}{p(x)}$ being uniformly continuous on $\mathbb{R}$"?

Tag (proof-writing) should not be the only tag a question has. Please add a tag for a subject area to which the question belongs. (from a bot)Normal Human 20 secs ago
0
Q: Center of a ring

1ENİGMA1Is there a prime ring $R$ such that $Z(R)=0$ ? There, $Z$ is a center. It is ohvious that If R is a commutative then there isn't prime ring R such that $Z(R)=0$,because $Z(R)=R$.How should we think for non-commutative ring ?

Short title. Center of a ring
0
Q: Algebra 2 & Matrix

Gracewhat does a small number on the base of a letter mean? I do not understand it and I am working on a problem in my Algebra 2 class that has that in it.

Short title. Short question. Algebra 2 & Matrix
0
Q: Branch of $\sqrt{1-z^2}$

TsangShow that a branch of $\sqrt{1-z^2}$ can be defined in any region $\Omega$ where the points $1,-1$ are in the same component of its complement. This is a question in Ahlfors' Complex Analysis (P.148 Q5) that I came across while trying to self-study the book. I tried to tackle the problem by cons...

0
Q: Closed form of $\sum\limits_{p = k}^{n - k + 1} C^{k - 1}_{p - 1} C^{k - 1}_{n - p}$

Nikita AstrakhantsevDoes anyone has an idea how to prove that $$\sum\limits_{p = k}^{n - k + 1} C^{k - 1}_{p - 1} C^{k - 1}_{n - p} = \frac{n!}{(2 k - 1)! (n - 2 k + 1)!}?$$

0
Q: How many orthogonal matrices(with a size of 3*3) are there, where all elements are whole numbers?

ThomaschenI know of course what is an orthogonal matrix, $Q^TQ=QQ^T=I$, where I is the identical matrix. Should I maybe start with checking the properties of the orthogonal matrices?

0
Q: need help in finding closed form for $\Sigma_{i=0}^{log(n/2)} \frac{n}{2^i}log\frac{n}{2^i}$

MaharajaXI need help in finding closed form for $$\Sigma_{i=0}^{log(n/2)} \frac{n}{2^i}log\frac{n}{2^i}$$ I am not sure even where to start. I know there is closed form for $$f(x) = \Sigma_{i=0}^\inf \frac{x}{2^i} = 2x$$, but this does not seem to help really... How would I find a closed form? Thanks

 
3:58 PM
-8
Q: This PHP topic must be undeleted

Shakti SinghPlease look at this question. this question has a lot of info for everyone and that is helpful. This question itself looks me a tutorial for every php programmer. I want to this question/discussion visible for everyone, guys, after all this is helpful for the community and that's why we are here...

 
0
Q: Panalphebetic strings provable by DFA?

VaroagIs the language of panalphabetic strings decidable by DFA? If so, how can I prove it?

 
00:00 - 16:0016:00 - 00:00

« first day (29 days earlier)      last day (545 days later) »